The notion that one might be justified in behaving irrationally in the service of a sufficiently worthy end is incohe...

doglvr on August 19, 2015

Help

Can u please explain why b is the answer and not a?

Replies
Create a free account to read and take part in forum discussions.

Already have an account? log in

Naz on September 4, 2015

Conclusion: the notion that one might be justified in behaving irrationally in the service of a sufficiently worthy end is incoherent.

Why? If an action is justified, then one would be behaving rationally.

P1: J ==> R
not R ==> not J

So the argument merely invokes the principle rule that if we are justified, then we are behaving rationally.

We are looking for the answer choice that is most similar in its reasoning to the argument. Let's start with answer choice (A):

(A): "A representative of the law, such as a judge or a police officer, ought not to commit crimes. For if representatives of the law commit crimes, they will be ineffective in preventing crime."

Conclusion: A representative of the law should not commit crimes.

Why? If a representative of the law commits a crime, then they will be ineffective in preventing crime.

P1: RLCC ==> not EPC
EPC ==> not RLCC

This clearly is not the same reasoning as in the argument in the passage since the argument in the passage states that "J" and "not R" occurring together is impossible because if we have "J," then we have to have "R." Answer choice (A) states that "RL"and "CC" occurring together is impossible because if we have "RLCC," then we have "not EPC."

Now let's look at answer choice (B):

"One cannot intend to spill a glass of water accidentally. Spilling it accidentally means that the act will not have been done intentionally."

Conclusion: "One cannot intend to spill a glass of water accidentally."

Why? If one spills a glass of water accidentally, then the act was not done intentionally.

P1: A ==> not I
I ==> not A

Answer choice (B) has the same reasoning as the argument in the passage since (B) is saying that "I" and "A" cannot occur together because if we have "A," then we have "not I."

Hope that clears things up! Please let us know if you have any other questions.

Ashley-Tien on July 2, 2018

Am I also supposed to be making sure that the language used in the stimulus matches up with that in the correct answer choices? I eliminated C and E immediately because they contained "both" while the stimulus didn't.

Mehran on July 3, 2018

Hi @Ashley-Tien. No. Sometimes the language will differ between stimulus & answer choice; sometimes it will be identical. Don't eliminate answer choices on parallel reasoning questions based on language alone. What matters is the logic.

Hope this helps! Please let us know if you have any additional questions.